Using stokes theorem to find magnetic field

Click For Summary
The discussion revolves around using Stokes' theorem to determine the magnetic field induced by a given current density. The magnetic field is expressed in azimuthal coordinates, and the current density is defined in terms of exponential functions. The user attempts to apply Stokes' theorem to relate the curl of the magnetic field to the current density, leading to an equation involving integrals over specified limits. However, there are concerns regarding the clarity of variables R, r, and p, as well as the surface of integration. The response indicates that the evaluation of the integral may be incorrect and emphasizes the need for clearer definitions and calculations.
indie452
Messages
115
Reaction score
0

Homework Statement



magnetic field is azimuthal B(r) = B(p,z) \phi
current density J(r) = Jp(p,z) p + Jz(p,z) z
= p*exp[-p] p + (p-2)*z*exp[-p] z

use stokes theorem to find B-filed induced by current everywhere in space

Homework Equations



stokes - {integral}dS.[curl A] = {closed integral}dl.A
curl B(r) = J(r)

The Attempt at a Solution



={integral}dS.[curl B(r)] = {closed integral}dl.B(r)
={integral}dS.J(r) = {closed integral}dl.B(p,z) \phi

{integral}dS.J(r) = {integral}dS.p*exp[-p] p + (p-2)*z*exp[-p] z = {closed integral}dl.B(p,z) \phi

dl = pd\phi \phi
dS = pd\phidz p + pd\phidp z

So:
{closed integral}pd\phi.B(p,z) \phi - with limits 0-2pi
= B(p,z)*2pi*p

{integral}dS.p*exp[-p] p + (p-2)*z*exp[-p] z
do in 2 parts:
{integral}pd\phidz.p*exp[-p] p - with limits 0-2pi and 0-R
= 2pi*p2*R*exp[-p]

{integral}pd\phidp.(p-2)*z*exp[-p] - with limits 0-2pi and 0-r
= -2pi*r2*exp[-r]

so B(p,z)*2pi*p = 2pi*p*R*exp[-p] - 2pi*r2*exp[-r]
= B(p,z) = p*R*exp[-p] - r2/p*exp[-r]

is this answer right?
 
Physics news on Phys.org
What's the difference between R, r, and p (by which I think you mean ρ)? That's the first thing you need to clear up.

You didn't say what surface S you're integrating over. In any case, I think your evaluation of

\oint_S (\nabla\times\textbf{B})\cdot d\textbf{S}

is incorrect.
 
Question: A clock's minute hand has length 4 and its hour hand has length 3. What is the distance between the tips at the moment when it is increasing most rapidly?(Putnam Exam Question) Answer: Making assumption that both the hands moves at constant angular velocities, the answer is ## \sqrt{7} .## But don't you think this assumption is somewhat doubtful and wrong?

Similar threads

  • · Replies 3 ·
Replies
3
Views
2K
Replies
6
Views
2K
  • · Replies 7 ·
Replies
7
Views
2K
  • · Replies 6 ·
Replies
6
Views
2K
  • · Replies 1 ·
Replies
1
Views
1K
  • · Replies 2 ·
Replies
2
Views
2K
  • · Replies 4 ·
Replies
4
Views
2K
Replies
4
Views
2K
  • · Replies 3 ·
Replies
3
Views
2K
Replies
9
Views
2K